Diễn Đàn MathScopeDiễn Đàn MathScope
  Diễn Đàn MathScope
Ghi Danh Hỏi/Ðáp Thành Viên Social Groups Lịch Ðánh Dấu Ðã Ðọc

Go Back   Diễn Đàn MathScope > Sơ Cấp > Việt Nam và IMO > 2011

News & Announcements

Ngoài một số quy định đã được nêu trong phần Quy định của Ghi Danh , mọi người tranh thủ bỏ ra 5 phút để đọc thêm một số Quy định sau để khỏi bị treo nick ở MathScope nhé !

* Nội quy MathScope.Org

* Một số quy định chung !

* Quy định về việc viết bài trong diễn đàn MathScope

* Nếu bạn muốn gia nhập đội ngũ BQT thì vui lòng tham gia tại đây

* Những câu hỏi thường gặp

* Về việc viết bài trong Box Đại học và Sau đại học


Trả lời Gởi Ðề Tài Mới
 
Ðiều Chỉnh Xếp Bài
Old 18-07-2011, 07:24 PM   #1
novae
+Thành Viên Danh Dự+
 
novae's Avatar
 
Tham gia ngày: Jul 2010
Đến từ: Event horizon
Bài gởi: 2,453
Thanks: 53
Thanked 3,057 Times in 1,288 Posts
Đề thi và đáp án IMO 2011

IMO 2011

Ngày 1


Bài 1. Cho một tập [M]A =\{a_{1}, a_{2}, a_{3}, a_{4}\}[/M] gồm 4 số nguyên dương phân biệt. Ký hiệu tổng [M]a_1+a_2+a_3+a_4[/M] bởi $s_A $. Đặt [M]n_A[/M] là số các cặp [M](i;j)[/M] với [M]1\leq i < j\leq 4[/M] và [M]a_i+a_j[/M] chia hết [M]s_A[/M]. Tìm tất cả các tập [M]A[/M] sao cho [M]n_A[/M] đạt giá trị lớn nhất có thể.

Bài 2. Một tập hữu hạn [M]S[/M] gồm ít nhất 2 điểm trên mặt phẳng. Giả sử không có 3 điểm nào của [M]S[/M] thẳng hàng. Một cối xay gió là một quá trình bắt đầu với một đường thẳng [M]\ell[/M] đi qua một điểm duy nhất [M]P\in S[/M]. Đường thẳng quay theo chiều kim đồng hồ quanh [M]P[/M] cho đến khi gặp một điểm khác cũng thuộc [M]S[/M]. Điểm mới này, [M]Q[/M], là trục quay mới, và đường thẳng [M]\ell[/M] tiếp tục quay theo chiều kim đồng hồ đến khi gặp một điểm khác của [M]S[/M]. Quá trình này lặp lại vô hạn lần.
Chứng minh rằng ta có thể chọn một điểm [M]P \in S[/M] và đường thẳng [M]\ell[/M] đi qua [M]P[/M] sao cho mỗi điểm của [M]S[/M] được sử dụng làm trục quay vô hạn lần.

Bài 3. Cho [M]f :\mathbb{R}\to\mathbb{R}[/M] thỏa mãn
[M]f(x+y)\leq yf(x)+f(f(x))[/M]
với mọi số thực [M]x,y[/M]. Chứng minh rằng [M]f(x)=0 \; \forall x \le 0[/M]

-------------------------------------------------------------------------


Ngày 2


Bài 4

Giả sử [M]n > 0[/M] là một số nguyên. Cho một cái cân hai đĩa và [M]n[/M] quả cân với trọng lượng là [M]{2^0}, {2^1}, ..., {2^{n - 1}}[/M]. Ta muốn đặt lên cái cân mỗi một trong [M]n[/M] quả cân, lần lượt từng quả một, theo cách để bảo đảm đĩa cân bên phải không bao giờ nặng hơn đĩa cân bên trái. Ở mỗi bước ta chọn một trong các quả cân chưa được đặt lên cân, rồi đặt nó hoặc vào đĩa bên trái, hoặc vào đĩa bên phải, cho đến khi tất cả các quả cân đều đã được đặt lên cân. Xác định xem có bao nhiêu cách để thực hiện được mục đích đề ra.

Bài 5

Cho hàm [M]f :\mathbb Z\rightarrow \mathbb Z_+[/M].Giả sử rằng với hai số nguyên [M]m, n[/M] bất kì, hiệu [M]f(m)-f(n )[/M] chia hết cho [M]f(m-n).[/M]
Chứng minh rằng với mọi số nguyên [M]m, n[/M] thỏa mãn [M]f(m)\leq f(n)[/M], thì ta có [M]f(n)[/M] chia hết cho [M]f(m)[/M]

Bài 6
Cho tam giác nhọn [M]ABC[/M] nội tiếp đường tròn [M]\Gamma[/M]. Gọi [M]l[/M] là tiếp tuyến tới [M]T[/M], và [M]l_a, l_bl, l_c[/M] là các đường thẳng đối xứng với [M]l[/M] qua [M]BC, CA, AB[/M] tương ứng.Chứng tỏ rằng đường tròn ngoại tiếp tam giác xác định bởi [M]l_a, l_b, l_c[/M] tiếp xúc với đường tròn [M]\Gamma[/M].

-----------------------------------------------------------------

Đề chính thức (Tiếng Việt): [Only registered and activated users can see links. ]
Đáp án tại trang cuối của topic.
[RIGHT][I][B]Nguồn: MathScope.ORG[/B][/I][/RIGHT]
 
File Kèm Theo
Kiểu File : pdf IMO2011.pdf (176.5 KB, 380 lần tải)
__________________
M.

thay đổi nội dung bởi: n.v.thanh, 05-08-2011 lúc 01:31 PM Lý do: Thêm đề chính thức.
novae is offline   Trả Lời Với Trích Dẫn
The Following 36 Users Say Thank You to novae For This Useful Post:
..... (19-07-2011), Anh Khoa (18-07-2011), AnhIsGod (17-05-2012), anh_96 (18-07-2011), bboy114crew (19-07-2011), chém gà (25-07-2011), conami (18-07-2011), crazy_nhox (18-07-2011), darkhole307 (18-07-2011), ghetvan (18-07-2011), ghost95 (22-07-2011), hizact (18-07-2011), hoangthang1607 (18-07-2011), hongson_vip (01-12-2012), huynhcongbang (19-07-2011), huynshin (20-07-2011), kaka_math (18-07-2011), kien10a1 (18-07-2011), kimlinh (18-07-2011), lexuanthang (18-07-2011), long_chau2010 (18-07-2011), MathForLife (18-07-2011), messiTLS (18-07-2011), n.v.thanh (18-07-2011), Ngô_Trung_Hiếu (19-07-2011), ngocson_dhsp (19-07-2011), nguyenhtctb (19-07-2011), nhat7d (19-07-2011), RiO (19-07-2011), thaipanh8 (13-08-2011), The Swastika (18-07-2011), tramanh (19-07-2011), tranbatphong (19-07-2011), trungthu10t (20-07-2011), Unknowing (18-07-2011), winwave (18-07-2011)
Old 18-07-2011, 07:38 PM   #2
NhamNgaHanh
Vọng Phong Nhi Đào
 
NhamNgaHanh's Avatar
 
Tham gia ngày: Jul 2011
Bài gởi: 282
Thanks: 85
Thanked 207 Times in 111 Posts
Bản pdf có link mathscope đi em.
[RIGHT][I][B]Nguồn: MathScope.ORG[/B][/I][/RIGHT]
 
__________________
Nhâm Ngã Hành
NhamNgaHanh is offline   Trả Lời Với Trích Dẫn
Old 18-07-2011, 07:47 PM   #3
n.v.thanh
Moderator
 
n.v.thanh's Avatar
 
Tham gia ngày: Nov 2009
Bài gởi: 2,849
Thanks: 2,980
Thanked 2,537 Times in 1,008 Posts
Nó trông na ná 2,3 bài đã từng xuất hiện rồi.Nhưng chăc là khó nhất ngày 1.

Bài 1. Ngon.
$s_A=a_1+a_2+a_3+a_4 $Có tất cả 6 cặp $a_i+a_j=a_1+a_2,a_2+a_3,a_3+a_4,a_1+a_4,a_2+a_3,a_ 2+a_4 $
Do 4 số nguyên dương phân biệt nên mình sắp lại thứ tự có $a_1<a_2<a_3<a_4 $.Khi đó $a_3+a_4|s_A $ khi $a_3+a_4|a_1+a_2 $ vô lý,cái $a_2+a_4 $ cũng thế.
Do vậy $n_A\leq 4 $
Có hệ $\begin{cases}
& a_1+a_2|s_A=a_1+a_2+a_3+a_4 \\
& a_1+a_3|s_A=a_1+a_2+a_3+a_4\\
& a_1+a_4|s_A=a_1+a_2+a_3+a_4\\
& a_2+a_3|s_A=a_1+a_2+a_3+a_4
\end{cases}
\rightarrow
\begin{cases}
& a_1+a_2|a_3+a_4 \\
& a_1+a_3|a_2+a_4\\
& a_1+a_4|a_2+a_3\\
& a_2+a_3|a_1+a_4
\end{cases}
\rightarrow
\begin{cases}
& a_1+a_2|a_3+a_4 \\
& a_1+a_3|a_2+a_4\\
& a_1+a_4=a_2+a_3\\
& a_1<a_2<a_3<a_4\\
\end{cases}
$

Cái này dễ giải ra tính được 4 số theo 1 số còn lại.Mathlinks ra đáp số ($x,5x,7x,11x $) và $(x,11x,19x,29x) $
[RIGHT][I][B]Nguồn: MathScope.ORG[/B][/I][/RIGHT]
 

thay đổi nội dung bởi: n.v.thanh, 18-07-2011 lúc 08:49 PM
n.v.thanh is offline   Trả Lời Với Trích Dẫn
The Following 2 Users Say Thank You to n.v.thanh For This Useful Post:
nhat7d (20-07-2011), thiendienduong (18-07-2011)
Old 18-07-2011, 07:57 PM   #4
novae
+Thành Viên Danh Dự+
 
novae's Avatar
 
Tham gia ngày: Jul 2010
Đến từ: Event horizon
Bài gởi: 2,453
Thanks: 53
Thanked 3,057 Times in 1,288 Posts
Trích:
Nguyên văn bởi crazy_nhox View Post
Bài 1 hình như Chép thiếu và nhầm rồi thì phải.
------------------------------
$s_{A} $ là tổng các phần tử của S, và $s_{a}\vdots a_{i}+a_{j} $ hay sao ấy chứ.
Đã bổ sung ký hiệu $s_A $.
$d|n \Leftrightarrow n \vdots d \Leftrightarrow n $ chia hết cho $d \Leftrightarrow d $ chia hết $n $
[RIGHT][I][B]Nguồn: MathScope.ORG[/B][/I][/RIGHT]
 
__________________
M.
novae is offline   Trả Lời Với Trích Dẫn
Old 18-07-2011, 08:23 PM   #5
eh_8g
+Thành Viên+
 
Tham gia ngày: Dec 2009
Bài gởi: 18
Thanks: 1
Thanked 17 Times in 10 Posts
Đóng góp bài 1:
Giải : Ta gọi Hai cặp trong là bù nhau nếu tổng bằng $s_a $.
Thấy rằng Hai cặp bù nhau, hoặc bằng nhau, hoặc chỉ có một trong chúng thuộc $n_a $.
Gs $a_1<a_2<a_3<a_4 $. Thì hoặc $a_1+a_2 $ hoặc $a_3+a_4 $ thuộc $n_a $.
$a_2+a_4 $ và $a_1+a_3 $ cũng vậy.
Do đó $n_a $ có tối đa 4 phần tử.
Điều này xảy ra khi : $a_1+a_4=a_2+a_3 $.
Đặt $a_1=x, a_2= x+d, a_3=y, a_4=y+d $.
Ta có : $a_1+a_2= 2x+d $ chia hết $2x+2y+2d $ (1)
$a_1+a_3= x+y $ chia hết $2d $ nhưng $y> x+d > d $. Nên $x+y =2d $.
(1) suy ra $2x+d $ chia hết $6d $. Vậy $2x+d=3d $ do $x<d $. Vậy $x= d:2 $.
Đáp số là $x, 3x, 5x, 7x $.
[RIGHT][I][B]Nguồn: MathScope.ORG[/B][/I][/RIGHT]
 

thay đổi nội dung bởi: novae, 18-07-2011 lúc 10:16 PM
eh_8g is offline   Trả Lời Với Trích Dẫn
Old 18-07-2011, 08:30 PM   #6
kien10a1
+Thành Viên+
 
kien10a1's Avatar
 
Tham gia ngày: Feb 2011
Đến từ: Vĩnh Yên- Vĩnh Phúc
Bài gởi: 371
Thanks: 43
Thanked 263 Times in 153 Posts
Gửi tin nhắn qua Yahoo chát tới kien10a1
Em thử bài 1. Ta hoàn toàn có thể giả sử $a_{1}<a_{2}<a_{3}<a_{4} $
Nhận xét $(a_{1}+a_{2}) +(a_{3}+a_{4})= s_{A}
(a_{2}+a_{3}) +(a_{1}+a_{4})= s_{A}
(a_{1}+a_{3}) +(a_{2}+a_{4})= s_{A} $
Do đó, trong 6 tổng $a_{i}+a_{j} $, có tối đa 4 tổng không vượt quá $\frac{n}{2} $, suy ra $n_{A}\leqslant 4 $
Nếu $n_{A}= 4 $, thì dễ thấy $(a_{2}+a_{3}) =(a_{1}+a_{4}) $
Lại có $(a_{1}+a_{3}) <(a_{2}+a_{4}), (a_{1}+a_{2}) <(a_{3}+a_{4}) $
Do đó $(a_{2}+a_{4})\vdots (a_{1}+a_{3}) $
Đặt $(a_{2}+a_{4})=k(a_{1}+a_{3}) $ , k nguyên dương, suy ra $2a_{2}=(k-1)a_{3}+ (k+1)a_{1} $ Mà $k\neq 1\Rightarrow k=2 $ ( do k <3)
Khi đó $(a_{3}+a_{4})\vdots (a_{1}+a_{2})\Leftrightarrow 5a_{3}+a_{1}\vdots a_{3}+5a_{1} $
Ta xét các trường hợp
TH1: $ 5a_{3}+a_{1}= 2(a_{3}+5a_{1}) $
TH2:$ 5a_{3}+a_{1}= 3(a_{3}+5a_{1}) $
TH3:$ 5a_{3}+a_{1}= 4(a_{3}+5a_{1}) $
TH1 không có nghiệm thỏa mãn.
TH2 : các số x,5x,7x,11x với x nguyên dương
TH3: y,11y,19y,29y với y nguyên dương
[RIGHT][I][B]Nguồn: MathScope.ORG[/B][/I][/RIGHT]
 
__________________
Quay về với nơi bắt đầu

thay đổi nội dung bởi: kien10a1, 18-07-2011 lúc 08:33 PM
kien10a1 is offline   Trả Lời Với Trích Dẫn
Old 18-07-2011, 09:33 PM   #7
hizact
+Thành Viên+
 
hizact's Avatar
 
Tham gia ngày: Apr 2010
Đến từ: Sài Gòn
Bài gởi: 535
Thanks: 287
Thanked 325 Times in 193 Posts
Mình mới soạn xong bản PDF đây: [Only registered and activated users can see links. ]
[RIGHT][I][B]Nguồn: MathScope.ORG[/B][/I][/RIGHT]
 

thay đổi nội dung bởi: hizact, 18-07-2011 lúc 09:43 PM
hizact is offline   Trả Lời Với Trích Dẫn
The Following 5 Users Say Thank You to hizact For This Useful Post:
kimlinh (18-07-2011), n.v.thanh (18-07-2011), phamtoan (01-08-2011), pHnAM (18-07-2011), thaipanh8 (13-08-2011)
Old 18-07-2011, 10:07 PM   #8
eh_8g
+Thành Viên+
 
Tham gia ngày: Dec 2009
Bài gởi: 18
Thanks: 1
Thanked 17 Times in 10 Posts
Bài 3:
[M]f(x+y) \le yf(x)+f(f(x))[/M] (1)
Thay [M]y=0[/M] ta có : [M]f(x) \le f(f(x))[/M]
Kí hiệu [M]D_f[/M] là tập giá trị của [M]f [/M]. Với [M]x \in D_f[/M], thay [M]y= f(x)-x[/M] không âm ta có [M]f(f(x)) \le yf(x)+f(f(x))[/M] nên [M]f(x)[/M] không âm với [M]x \in D_f[/M] nói cách khác [M]f(f(x))[/M] không âm với mọi [M]x [/M].
Bây giờ, giả sử tồn tại [M]x[/M] mà [M]f(x)>0 [/M], cho y tiến tới âm vô cùng thì sẽ tồn tại [M]e[/M] mà [M]f(x)<0[/M] với mọi [M]x \le e[/M]
tương tự nếu có [M]f(x)<0[/M] thì [M]f(x)<0[/M] với mọi [M]x[/M] đủ lớn.
Chú ý rằng nếu [M]f(x)<0[/M] thì [M]x[/M] không thuộc tập giá trị của [M]f[/M]
TH1: [M]f[/M] nhận cả giá trị âm dương. Thì [M]D_f[/M] là tập bị chặn . Mâu thuẫn.
TH2: [M]f[/M] chỉ nhận giá trị dương cũng mâu thuẫn
Vậy [M]f[/M] chỉ nhận giá trị không dương .
Như thế [M]f(f(x))=0[/M] với mọi [M]x[/M]
suy ra [M]f(0)=f(f(f(x)))=0[/M]
Ta có [M]f(x+y) \le yf(x)[/M]
Cho [M]y=-x[/M] suy ra [M]-xf(x) \ge 0[/M] nên [M]f(x)[/M] không âm với [M]x[/M] không âm
Suy ra [M]f(x)=0[/M] với [M]x[/M] không âm.
[RIGHT][I][B]Nguồn: MathScope.ORG[/B][/I][/RIGHT]
 

thay đổi nội dung bởi: novae, 18-07-2011 lúc 10:12 PM
eh_8g is offline   Trả Lời Với Trích Dẫn
Old 18-07-2011, 10:19 PM   #9
pgviethung
+Thành Viên+
 
Tham gia ngày: Sep 2010
Bài gởi: 142
Thanks: 1
Thanked 68 Times in 54 Posts
Nếu $y=f(x)-x=0 $ thì ta chưa thể kết luận là $f(f(x)) $ không âm.
[RIGHT][I][B]Nguồn: MathScope.ORG[/B][/I][/RIGHT]
 
pgviethung is offline   Trả Lời Với Trích Dẫn
Old 19-07-2011, 01:34 AM   #10
pgviethung
+Thành Viên+
 
Tham gia ngày: Sep 2010
Bài gởi: 142
Thanks: 1
Thanked 68 Times in 54 Posts
Bài 3: $f(x+y) \leq yf(x)+f(f(x)). $ (*)
+ Cho $y=0 $, có $f(x)\leq f(f(x)) \; $ (1).
+ Cho $y=f(x)-x $, có $f(f(x)) \leq f(x)[f(x)-x]+f(f(x)) $, hay là $0 \leq f(x)[f(x)-x] \; $ (2).
a) Ta sẽ chứng minh $f(x)\leq 0 $ với mọi $x $. Thật vậy, nếu có $x $ sao cho $f(x)>0 $ thì
$f(f(y))=f(f(y)-x+x) \leq [f(y)-x]f(x)+f(f(x)) = [f(y-x+x)-x]f(x)+f(f(x)) \leq [(y-x)f(x)+f(f(x))-x]f(x)+f(f(x)). $
Ta có $f(f(y))\leq y.f^2(x)+h(x) $ với mọi $y $ (3) ($h(x) $ là hàm xác định từ biểu thức trên).
Chú ý là: từ (1) và (2), ta thấy $0 \leq f(f(x))[f(f(x))-f(x)] $ và $f(x)\leq f(f(x)) $, nên nếu $f(f(x))<0 $ thì phải có $f(f(x))=f(x) $.
Như vậy, từ (3) suy ra: khi $y $ tới âm vô cùng thì $f(y)=f(f(y)) $ cũng tới âm vô cùng.
Thay $y=-x $ vào (*), có $f(0)\leq -xf(x)+f(f(x))=f(x)(-x+1) $ (khi $x $ đủ nhỏ thì $f(x)=f(f(x)) $). Cho $x $ tới âm vô cùng thì vế phải cũng tới âm vô cùng, trong khi vế trái cố định. Mâu thuẫn.
b) Ta chứng minh rằng tồn tại N để với mọi $x<N $ thì $f(x)=0. $
Phản chứng, nếu không tồn tại N thì từ ý a), ta suy ra tồn tại dãy $\{x_n\} $ dần tới âm vô cùng sao cho $f(x_n)<0 $. Như vậy, từ (2) ta suy ra $f(x)-x\leq 0 $.
Lại thay $y=-x $ vào (*), có $f(0) \leq -xf(x)+f(f(x)) \leq -xf(x) $. Chọn dãy $\{x_n\} $ tiến tới âm vô cùng thay vào, ta có vế trái cố định, vế phải ra âm vô cùng. Mâu thuẫn.
c) Xét $x<N $, ta có $0 \geq f(0)=f(f(x)) \geq f(x) =0 $. Vậy $f(0)=0 $.
d) Xét $x<0 $, có $0=f(0)=f(-x+x) \leq -xf(x)+f(f(x))\leq -xf(x) \leq 0 $. Dấu bằng xảy ra khi $f(x)=0 $.
KL: $f(x)=0 $ với mọi $x\leq 0 $.
[RIGHT][I][B]Nguồn: MathScope.ORG[/B][/I][/RIGHT]
 

thay đổi nội dung bởi: pgviethung, 19-07-2011 lúc 01:41 AM
pgviethung is offline   Trả Lời Với Trích Dẫn
Old 19-07-2011, 02:20 AM   #11
Traum
Moderator
 
Traum's Avatar
 
Tham gia ngày: Nov 2007
Đến từ: cyber world
Bài gởi: 413
Thanks: 14
Thanked 466 Times in 171 Posts
Bài 2:
Ta gọi một đường thằng $d $ đi qua điểm $A $ và một điểm $B $ là tốt với $A $ nếu quay $d $ quanh $A $ một góc $\alpha $ nhỏ (chiều quay kim đồng hồ) thì hai nửa mặt phẳng chứa số điểm chênh lệch nhau không quá $1 $.

Nhận xét 1: Mỗi điểm $A $ thì có ít nhất $1 $ đường thẳng tốt đi qua.

Bây giờ ta sẽ chứng minh là với cách chọn điểm $A $ bất kì và đường thẳng $ l $ là đường thẳng tốt qua $A $

Nhận xét 2: Giả sử tại bước thứ $k $, ta quay tại điểm $A $ và đường thẳng $l $ là đường thẳng tốt thì ta sẽ đến tới điểm $B $, đường thẳng $l $ sẽ trở thành đường thẳng $BA $ và $BA $ là tốt
Chứng minh:

Thật vậy ta giả sử $l $ chứa $2 $ điểm $A $ và $X $( song song với đường ngang). Số điểm nằm ở mặt phẳng phía trên là $a $, phía dưới là $b $. Ta có do $l $ tốt nên $|a+1-b|\le 1 $. Nếu $B $ ở mặt phẳng phía trên thì khi quay đường $BA $ một góc $\alpha $ nhỏ chia mặt phẳng thành hai phần có số điểm chênh nhau là $|a+1-b|\le 1 $. Nếu $B $ ở phía dưới thì cũng dễ thấy khi quay $BA $ quanh $B $ một góc $\alpha $ nhỏ đi thì số điểm chênh lệch ở hai nửa mặt phẳng là $|a+1-b|\le 1 $. Do đó $BA $ là đường thẳng tốt. Nhận xét được chứng minh.

Nhận xét 3: Mỗi điểm $X $ thuộc $S $ sẽ được ghé đến ít nhất một lần không phụ thuộc vào điểm đầu ( và do đó được ghé đến vô hạn lần).

Ta biết rằng quá trình quay đường thẳng $l $ theo chiều kim đồng hồ là một quá trình vô hạn lần và sẽ có lúc đường thẳng $l $ song song với đường thẳng tốt $d $ đi qua $X $ và điểm $M $. Nếu $l $ không trùng $d $, xét điểm cuối cùng mà $l $ đi qua trước khi song song với $X $ là $N,Y $. Dễ thấy là $M,X $ khác với $N,Y $Tuy nhiên dẽ thấy là sẽ có ít nhất một trong hai đường thẳng $NY $ và $MX $ không tốt. ( chỉ cần so sánh số điểm chênh lệch ở các nửa mặt phẳng là ok)

Vậy bài toán được chứng minh
[RIGHT][I][B]Nguồn: MathScope.ORG[/B][/I][/RIGHT]
 
__________________
Traum is giấc mơ.

thay đổi nội dung bởi: Traum, 19-07-2011 lúc 02:27 AM
Traum is offline   Trả Lời Với Trích Dẫn
The Following User Says Thank You to Traum For This Useful Post:
huynhcongbang (19-07-2011)
Old 19-07-2011, 06:23 AM   #12
batigoal
Super Moderator
 
batigoal's Avatar
 
Tham gia ngày: Jul 2010
Đến từ: Hà Nội
Bài gởi: 2,895
Thanks: 382
Thanked 2,968 Times in 1,295 Posts
Đây là đề bài và lời giải IMO 2011. Bản latex.pdf
[RIGHT][I][B]Nguồn: MathScope.ORG[/B][/I][/RIGHT]
 
File Kèm Theo
Kiểu File : pdf DE BAI VA LOI GIAI IMO 2011.pdf (112.7 KB, 1383 lần tải)
__________________
“ Sức mạnh của tri thức là sự chia sẻ tri thức”

[Only registered and activated users can see links. ]
batigoal is offline   Trả Lời Với Trích Dẫn
The Following 6 Users Say Thank You to batigoal For This Useful Post:
chemmath (19-07-2011), ghetvan (19-07-2011), kimlinh (20-07-2011), minhgiao (21-08-2011), pHnAM (19-07-2011), thaipanh8 (13-08-2011)
Old 19-07-2011, 07:52 AM   #13
toanlc_gift
+Thành Viên+
 
toanlc_gift's Avatar
 
Tham gia ngày: Jan 2009
Đến từ: FU
Bài gởi: 171
Thanks: 31
Thanked 142 Times in 80 Posts
Trích:
Nguyên văn bởi pgviethung View Post
Bài 3: $f(x+y) \leq yf(x)+f(f(x)). $ (*)
+ Cho $y=0 $, có $f(x)\leq f(f(x)) \; $ (1).
+ Cho $y=f(x)-x $, có $f(f(x)) \leq f(x)[f(x)-x]+f(f(x)) $, hay là $0 \leq f(x)[f(x)-x] \; $ (2).
a) Ta sẽ chứng minh $f(x)\leq 0 $ với mọi $x $. Thật vậy, nếu có $x $ sao cho $f(x)>0 $ thì
$f(f(y))=f(f(y)-x+x) \leq [f(y)-x]f(x)+f(f(x)) = [f(y-x+x)-x]f(x)+f(f(x)) \leq [(y-x)f(x)+f(f(x))-x]f(x)+f(f(x)). $
Ta có $f(f(y))\leq y.f^2(x)+h(x) $ với mọi $y $ (3) ($h(x) $ là hàm xác định từ biểu thức trên).
Chú ý là: từ (1) và (2), ta thấy $0 \leq f(f(x))[f(f(x))-f(x)] $ và $f(x)\leq f(f(x)) $, nên nếu $f(f(x))<0 $ thì phải có $f(f(x))=f(x) $.
Như vậy, từ (3) suy ra: khi $y $ tới âm vô cùng thì $f(y)=f(f(y)) $ cũng tới âm vô cùng.
Thay $y=-x $ vào (*), có $f(0)\leq -xf(x)+f(f(x))=f(x)(-x+1) $ (khi $x $ đủ nhỏ thì $f(x)=f(f(x)) $). Cho $x $ tới âm vô cùng thì vế phải cũng tới âm vô cùng, trong khi vế trái cố định. Mâu thuẫn.
b) Ta chứng minh rằng tồn tại N để với mọi $x<N $ thì $f(x)=0. $
Phản chứng, nếu không tồn tại N thì từ ý a), ta suy ra tồn tại dãy $\{x_n\} $ dần tới âm vô cùng sao cho $f(x_n)<0 $. Như vậy, từ (2) ta suy ra $f(x)-x\leq 0 $.
Lại thay $y=-x $ vào (*), có $f(0) \leq -xf(x)+f(f(x)) \leq -xf(x) $. Chọn dãy $\{x_n\} $ tiến tới âm vô cùng thay vào, ta có vế trái cố định, vế phải ra âm vô cùng. Mâu thuẫn.
c) Xét $x<N $, ta có $0 \geq f(0)=f(f(x)) \geq f(x) =0 $. Vậy $f(0)=0 $.
d) Xét $x<0 $, có $0=f(0)=f(-x+x) \leq -xf(x)+f(f(x))\leq -xf(x) \leq 0 $. Dấu bằng xảy ra khi $f(x)=0 $.
KL: $f(x)=0 $ với mọi $x\leq 0 $.
Lời giải này của bạn không ổn rồi
Trích:
Ta sẽ chứng minh $f(x)\leq 0 $ với mọi $x $. Thật vậy, nếu có $x $ sao cho $f(x)>0 $
ở chỗ này bạn đang giả sử tồn tại một vài giá trị cụ thể của x,lúc này x có thể coi là hằng số,không thể cho chạy x được,nhưng ở đoạn sau lại có
Trích:
Cho $x $ tới âm vô cùng thì vế phải cũng tới âm vô cùng, trong khi vế trái cố định. Mâu thuẫn.
bạn mới chỉ chứng minh được tồn tại ${x_0} $ sao cho
$f(0) \le (1 - {x_0})f({x_0}) $
cả hai vế đều là hằng số thì chưa giải quyết được gì đâu.
-------------------------------------------------------
p/s:bài này khó thế =.=
[RIGHT][I][B]Nguồn: MathScope.ORG[/B][/I][/RIGHT]
 
toanlc_gift is offline   Trả Lời Với Trích Dẫn
Old 19-07-2011, 10:42 AM   #14
huynhcongbang
Administrator

 
huynhcongbang's Avatar
 
Tham gia ngày: Feb 2009
Đến từ: Ho Chi Minh City
Bài gởi: 2,413
Thanks: 2,165
Thanked 4,188 Times in 1,381 Posts
Gửi tin nhắn qua Yahoo chát tới huynhcongbang
Trích:
Nguyên văn bởi Traum View Post
Nhận xét 2: Giả sử tại bước thứ $k $, ta quay tại điểm $A $ và đường thẳng $l $ là đường thẳng tốt thì ta sẽ đến tới điểm $B $, đường thẳng $l $ sẽ trở thành đường thẳng $BA $ và $BA $ là tốt
Chứng minh:

Thật vậy ta giả sử $l $ chứa $2 $ điểm $A $ và $X $( song song với đường ngang). Số điểm nằm ở mặt phẳng phía trên là $a $, phía dưới là $b $. Ta có do $l $ tốt nên $|a+1-b|\le 1 $. Nếu $B $ ở mặt phẳng phía trên thì khi quay đường $BA $ một góc $\alpha $ nhỏ chia mặt phẳng thành hai phần có số điểm chênh nhau là $|a+1-b|\le 1 $. Nếu $B $ ở phía dưới thì cũng dễ thấy khi quay $BA $ quanh $B $ một góc $\alpha $ nhỏ đi thì số điểm chênh lệch ở hai nửa mặt phẳng là $|a+1-b|\le 1 $. Do đó $BA $ là đường thẳng tốt. Nhận xét được chứng minh.
Cho em hỏi về nhận xét 2 này:
Nếu theo cách định nghĩa đường thẳng tốt thì BĐT phải là $|a-b| \le 1 $.
Đường thẳng $\l $ ban đầu tuy chỉ đi qua duy nhất 1 điểm thuộc S nhưng khi quay nhiều lần thì theo đề bài, chưa hẳn là đi qua chỉ 1 điểm.
Giả sử trong nhận xét 2 trên, đường thẳng $\l $ là đường thẳng tốt với A và tại thời điểm đó nó đi qua nhiều điểm nữa khác A, giả sử là $k , k \ge 2 $ điểm nữa và khi quay nó tới B theo chiều kim đồng hồ thì đường thẳng mới chỉ chứa 2 điểm A, B và một trong hai nửa mặt phẳng chia ra bởi AB nhận toàn bộ k điểm nêu trên thì số điểm giữa hai nửa mp không còn chênh lệch không quá 1 nữa.
[RIGHT][I][B]Nguồn: MathScope.ORG[/B][/I][/RIGHT]
 
huynhcongbang is offline   Trả Lời Với Trích Dẫn
Old 19-07-2011, 11:27 AM   #15
kien10a1
+Thành Viên+
 
kien10a1's Avatar
 
Tham gia ngày: Feb 2011
Đến từ: Vĩnh Yên- Vĩnh Phúc
Bài gởi: 371
Thanks: 43
Thanked 263 Times in 153 Posts
Gửi tin nhắn qua Yahoo chát tới kien10a1
Trích:
Nguyên văn bởi huynhcongbang View Post
Cho em hỏi về nhận xét 2 này:
Nếu theo cách định nghĩa đường thẳng tốt thì BĐT phải là $|a-b| \le 1 $.
Đường thẳng $\l $ ban đầu tuy chỉ đi qua duy nhất 1 điểm thuộc S nhưng khi quay nhiều lần thì theo đề bài, chưa hẳn là đi qua chỉ 1 điểm.
Giả sử trong nhận xét 2 trên, đường thẳng $\l $ là đường thẳng tốt với A và tại thời điểm đó nó đi qua nhiều điểm nữa khác A, giả sử là $k , k \ge 2 $ điểm nữa và khi quay nó tới B theo chiều kim đồng hồ thì đường thẳng mới chỉ chứa 2 điểm A, B và một trong hai nửa mặt phẳng chia ra bởi AB nhận toàn bộ k điểm nêu trên thì số điểm giữa hai nửa mp không còn chênh lệch không quá 1 nữa.
Em nghĩ là nên thế này, ta sẽ không nhắc đến đường thẳng AB( vì hiệu của ta nói đến có thể là 2 ), ta thay nhận xét là, khi đường thẳng quay từ A đến B, và khi nó dời trục quay sang B, và không đi qua A nữa thì số điểm ở 2 bên mặt phẳng vẫn là bất biến( thật vậy, một nửa mp mất B, được A, một nửa mặt phẳng giữ nguyên số điểm)
[RIGHT][I][B]Nguồn: MathScope.ORG[/B][/I][/RIGHT]
 
__________________
Quay về với nơi bắt đầu
kien10a1 is offline   Trả Lời Với Trích Dẫn
The Following User Says Thank You to kien10a1 For This Useful Post:
Traum (19-07-2011)
Trả lời Gởi Ðề Tài Mới

Bookmarks

Ðiều Chỉnh
Xếp Bài

Quuyền Hạn Của Bạn
You may not post new threads
You may not post replies
You may not post attachments
You may not edit your posts

BB code is Mở
Smilies đang Mở
[IMG] đang Mở
HTML đang Tắt

Chuyển đến


Múi giờ GMT. Hiện tại là 01:02 AM.


Powered by: vBulletin Copyright ©2000-2024, Jelsoft Enterprises Ltd.
Inactive Reminders By mathscope.org
[page compression: 118.45 k/135.06 k (12.30%)]